Non Homogeneous Recurrence Relation

Click For Summary
The discussion revolves around solving the non-homogeneous recurrence relation (n+2)a[n+1] = 2(n+1)a[n] + 2^n with the initial condition a[0] = 1. The user has manipulated the equation by shifting indices and eliminating the 2^n term, leading to a new equation involving terms (n+1), n, and (n-1). They express uncertainty about how to handle these terms when determining the particular and homogeneous solutions. A substitution was suggested, leading to a simplified form of the recurrence, but the user remains unsure about the next steps and how to incorporate the initial condition effectively. The conversation highlights the complexities of solving non-homogeneous recurrence relations and the need for clarity in the substitution process.
Ethers0n
Messages
27
Reaction score
0
1. solve the following recurrence relation for an



2. (n+2)an+1= 2(n+1)an+2^{n}, n>=0, a0=1
I shifted the index, multiplied through by the 2^{n} term and then subtracted the resulting equation from the original equation to get rid of the 2^{n} term...


3. I have gotten to this point
(n+1)an-4(n)an-14(n-1)an-2=0


I'm not really sure how to handle the (n+1), n, or (n-1) terms when looking for the particular/ homogeneous solution parts.
 
Physics news on Phys.org
can you see a way to change your variable a(n) that would do it? There is something consistemt between the successive terms.

(You have missed a + out of your formula BTW.)
 
well, if I sub in bm = (n+1)an into the original equation of
(n+2)an+1 = 2(n+1)an+2n
I get
bm+1=2bm+2m
(1) bm+1-2bm=2m
(2) bm-2bm-1=2m-1
(3) 2bm-4bm-1=2m
(1)-(3)
(4) bm+1-4bm+4bm-1=0
(5) bm-4bm-1+4bm-2=0
(6) r2-4r+4=0
(7) (r-2)(r-2)=0
(8) bm= c12m+c2m2m

but I don't really know where to go from there?
do I sub back in, or is there a way to use a0=1 with bm?
I'm getting the feeling that I'm dong something wrong...
 
Question: A clock's minute hand has length 4 and its hour hand has length 3. What is the distance between the tips at the moment when it is increasing most rapidly?(Putnam Exam Question) Answer: Making assumption that both the hands moves at constant angular velocities, the answer is ## \sqrt{7} .## But don't you think this assumption is somewhat doubtful and wrong?

Similar threads

Replies
8
Views
3K
  • · Replies 15 ·
Replies
15
Views
3K
  • · Replies 11 ·
Replies
11
Views
2K
  • · Replies 6 ·
Replies
6
Views
2K
  • · Replies 2 ·
Replies
2
Views
2K
  • · Replies 1 ·
Replies
1
Views
2K
  • · Replies 1 ·
Replies
1
Views
2K
  • · Replies 7 ·
Replies
7
Views
2K
  • · Replies 5 ·
Replies
5
Views
2K
  • · Replies 4 ·
Replies
4
Views
1K